12) Models of Set Theory - Suggested Solutions to Jech's Set Theory

$ \newcommand{\func}{\operatorname} \newcommand{\lcm}{\func{lcm}} \newcommand{\hl}[1]{\textbf{#1}} \newcommand{\lf}{\lfloor} \newcommand{\rf}{\rfloor} \newcommand{\f}[1]{\left\lf #1 \right\rf} \newcommand{\bb}[1]{\left( #1 \right)} \newcommand{\Sb}[1]{\left\lbrack #1 \right\rbrack} \renewcommand{\ss}[1]{\left\lbrace #1 \right\rbrace} \renewcommand{\mod}[1]{\left\lvert #1 \rightrvert} \newcommand{\R}{\boldsymbol{R}} \newcommand{\C}{\boldsymbol{C}} \newcommand{\Z}{\boldsymbol{Z}} \newcommand{\Q}{\boldsymbol{Q}} \renewcommand{\P}{\mathbb{P}} \newcommand{\Po}{\mathcal{P}} \newcommand{\F}{\mathcal{F}} \newcommand{\Zz}{\Z_{\geq 0}} \renewcommand{\c}[1]{\left\langle #1 \right\rangle} \renewcommand{\mod}[1]{\left\lvert #1 \right\rvert} \newcommand{\norm}[1]{\left\lVert #1 \right\rVert} \renewcommand{\bar}[1]{\overline{#1}} \newcommand{\eqbreak}{\phantom{{} = {}}} \newcommand{\impliesbreak}{\phantom{{} \implies {}}} \newcommand{\iffbreak}{\phantom{{} \iff {}}} \newcommand{\del}{\partial} \renewcommand{\d}[1]{\, \mathrm{d} #1} \newcommand{\dydt}{\frac{\d{y}}{\d{t}}} \newcommand{\dydx}{\frac{\d{y}}{\d{x}}} \newcommand{\pd}[2]{\frac{\del #1}{\del #2}} \newcommand{\pp}[3]{\frac{\del^2 #1}{\del #2 \del #3}} \newcommand{\dd}[2]{\frac{\mathrm{d} #1}{\mathrm{d} #2}} \newcommand{\Inn}{\func{Inn}} \newcommand{\Aut}{\func{Aut}} \newcommand{\diag}{\func{diag}} \newcommand{\id}{\func{id}} \newcommand{\GL}{\func{GL}} \newcommand{\SL}{\func{SL}} \newcommand{\End}{\func{End}} \newcommand{\Hom}{\func{Hom}} \newcommand{\Tr}{\func{Tr}} \newcommand{\rank}{\func{rank}} \newcommand{\ORD}{\mathbf{ORD}} \newcommand{\OD}{\mathbf{OD}} \newcommand{\HOD}{\mathbf{HOD}} \newcommand{\ZF}{\mathsf{ZF}} \newcommand{\ZFC}{\mathsf{ZFC}} \newcommand{\ZC}{\mathsf{ZC}} \renewcommand{\AC}{\mathsf{AC}} \newcommand{\DC}{\mathsf{DC}} \newcommand{\CC}{\mathsf{CC}} \newcommand{\CH}{\mathsf{CH}} \newcommand{\SH}{\mathsf{SH}} \newcommand{\KH}{\mathsf{KH}} \newcommand{\GCH}{\mathsf{GCH}} \newcommand{\PA}{\mathsf{PA}} \newcommand{\BST}{\mathsf{BST}} \newcommand{\MA}{\mathsf{MA}} \newcommand{\Con}{\func{Con}} \newcommand{\ON}{\mathbf{ON}} \newcommand{\dom}{\func{dom}} \newcommand{\ran}{\func{ran}} \newcommand{\pred}{\func{pred}} \newcommand{\mos}{\func{mos}} \newcommand{\WF}{\mathbf{WF}} \newcommand{\type}{\func{type}} \newcommand{\V}{\mathbf{V}} \renewcommand{\L}{\mathcal{L}} \newcommand{\cl}{\func{cl}} \newcommand{\trcl}{\func{tc}} \newcommand{\TC}{\func{TC}} \newcommand{\A}{\mathfrak{A}} \newcommand{\B}{\mathfrak{B}} \newcommand{\add}{\func{add}} \newcommand{\cov}{\func{cov}} \newcommand{\non}{\func{non}} \newcommand{\cf}{\func{cf}} \newcommand{\forces}{\Vdash} \newcommand{\one}{\mathbbm{1}} \newcommand{\restrictedto}{\mathord{\upharpoonright}} \newcommand{\Fn}{\func{Fn}} \newcommand{\Col}{\func{Col}} \newcommand{\bigtriangle}{\triangle} \newcommand{\U}{\mathcal{U}} \newcommand{\W}{\mathcal{W}} \newcommand{\N}{\mathcal{N}} \newcommand{\Seq}{\mathit{Seq}} \newcommand{\lh}{\func{lh}} \newcommand{\Ult}{\func{Ult}} \newcommand{\In}{\func{In}} \newcommand{\otp}{\func{otp}} \newcommand{\symdiff}{\, \triangle \,} \newcommand{\crit}{\func{crit}} \newcommand{\sat}{\func{sat}} \renewcommand{\int}{\func{int}} \newcommand{\length}{\func{length}} \newcommand{\Lim}{\func{Lim}} \newcommand{\Succ}{\func{Succ}} \newcommand{\G}{\mathcal{G}} \newcommand{\T}{\mathcal{T}} \newcommand{\ext}{\func{ext}} \newcommand{\height}{\func{height}} \renewcommand{\diamond}{\diamondsuit} \newcommand{\D}{\mathcal{D}} \newcommand{\NS}{\mathrm{NS}} $


Exercise 12.1.

Solution. Step 1 - Construct isomorphism $f$: We define $f : \A_a \to \Ult_U\lbrace \A_x : x \in S\rbrace$ as follows: Fix any $y_x \in \A_x$ for each $x \in S - \lbrace a\rbrace$. For each $y \in \A_a$, define $g_y : S \to \prod_{x \in S} \A_x$ by stipulating that:
$$ \begin{align*}
g_y(x) =
\begin{cases}
y, &\text{if $x = a$} \\
y_x, &\text{otherwise}
\end{cases}
\end{align*}$$ Finally, we may define $f$ by:
$$ \begin{align*}
f(y) = [g_y]
\end{align*}$$ We now shall that $f$ is an isomorphism.

Step 2 - Prove $f$ is bijective: We first see that it injective, as:
$$ \begin{align*}
f(y) = f(y') &\implies [g_y] = [g_{y'}] \\
&\implies \lbrace x \in S : g_y(x) = g_{y'}(x)\rbrace \in U && \text{by Łoś Theorem} \\
&\implies g_y(a) = g_{y'}(a) \\
&\implies y = y'
\end{align*}$$ It is also surjective, as for any $h : S \to \prod_{x \in S} \A_x$, we have $[h] = [g_{h(a)}]$, since:
$$ \begin{align*}
h(a) = g_{h(a)}(a) \implies \lbrace x \in S : h(x) = g_{h(a)}(x)\rbrace \in U \implies [h] = [g_{h(a)}]
\end{align*}$$ Step 3 - Prove $f$ is elementary: We now show that it respects predicates, functions and constants. Let $P$ be a predicate symbol. Then:
$$ \begin{align*}
P^{\A}(f(x_1),\dots,f(x_1)) &\iff P^{\A}([g_{x_1}],\dots,[g_{x_n}]) \\
&\iff \lbrace x \in S : P^{\A_x}(g_{x_1}(x),\dots,g_{x_n}(x))\rbrace \in U \\
&\iff P^{\A_a}(g_{x_1}(a),\dots,g_{x_n}(a)) \\
&\iff P^{\A_a}(x_1,\dots,x_n)
\end{align*}$$ If $F$ is a function symbol, then:
$$ \begin{align*}
F^{\A}(f(x_1),\dots,f(x_n)) &= F^{\A}([g_{x_1}],\dots,[g_{x_n}]) \\
&= [x \mapsto F^{\A_x}(g_{x_1}(x),\dots,g_{x_1}(x))] \\
&= \lbrace f : f(a) = F^{\A_a}(g_{x_1}(a),\dots,g_{x_n}(a))\rbrace \\
&= \lbrace f : f(a) = F^{\A_a}(x_1,\dots,x_n)\rbrace \\
&= [g_{F^{\A_a}(x_1,\dots,x_n)}] \\
&= f(F^{\A_a}(x_1,\dots,x_n))
\end{align*}$$ Finally, if $c$ is a constant symbol, then:
$$ \begin{align*}
c^{\A} = [x \mapsto c^{\A_x}] = \lbrace f : f(a) = c^{\A_a}\rbrace = [g_{c^{\A_a}}] = f(c^{\A_a})
\end{align*}$$

$\square$




Exercise 12.2.

Solution. Suppose $U$ is a principal ultrafilter on the set $S$.

Step 1 - Prove that $j$ is bijective: We see that it is injective, as:
$$ \begin{align*}
j(x) = j(x') &\implies [c_x] = [c_{x'}] \\
&\implies \lbrace y \in S : c_x(y) = c_{x'}(y)\rbrace \in U \\
&\implies \lbrace y \in S : x = x'\rbrace \in U \\
&\implies \lbrace y \in S : x = x'\rbrace \neq \emptyset \\
&\implies x = x'
\end{align*}$$ It is surjective, as for any $f : S \to A$, we have $[f] = [c_{f(a)}]$.

Step 2 - Prove that $j$ is elementary: We now show that it respects predicates, functions and constants. Let $P$ be a predicate symbol. Then:
$$ \begin{align*}
P^{\Ult_U\A}(j(x_1),\dots,j(x_1)) &\iff P^{\Ult_U\A}([c_{x_1}],\dots,[c_{x_n}]) \\
&\iff \lbrace y \in S : P^{\A}(c_{x_1}(y),\dots,g_{x_n}(y))\rbrace \in U \\
&\iff \lbrace y \in S : P^{\A}(x_1,\dots,x_n)\rbrace \in U \\
&\iff P^{\A}(x_1,\dots,x_n)
\end{align*}$$ The proof for function and constant symbols are similar.

$\square$




Exercise 12.3.


Exercise 12.3(i).

Solution. Let $f,g,h : \kappa \to \kappa$ be three functions. Clearly $[f] \not<^\ast [f]$, as $\lbrace \alpha \in \kappa : f(\alpha) < f(\alpha)\rbrace = \emptyset \notin U$. Since:
$$ \begin{align*}
[f] <^\ast [g] &\iff \lbrace \alpha \in \kappa : f(\alpha) < g(\alpha)\rbrace \in U \\
[g] <^\ast [h] &\iff \lbrace \alpha \in \kappa : g(\alpha) < h(\alpha)\rbrace \in U
\end{align*}$$ We have that, if $[f] <^\ast [g]$ and $[g] <^\ast [h]$ then:
$$ \begin{align*}
\lbrace \alpha \in \kappa : f(\alpha) < h(\alpha)\rbrace \subseteq \lbrace \alpha \in \kappa : f(\alpha) < g(\alpha)\rbrace \cap \lbrace \alpha \in \kappa : g(\alpha) < h(\alpha)\rbrace \in U
\end{align*}$$ so $[f] <^\ast [h]$ as well. Hence, $<^\ast $ is a partial ordering.

To see that it is a linear ordering, define:
$$ \begin{align*}
X_R := \lbrace \alpha \in \kappa : f(\alpha) \; R \; g(\alpha)\rbrace
\end{align*}$$ where $R \in \lbrace =,<,>\rbrace$. Clearly $\kappa = X_< \sqcup X_= \sqcup X_>$. Thus, we must have that exactly one of $X_<,X_=,X_>$ is in $U$, which corresponds to $[f] <^\ast [g]$, $[f] =^\ast [g]$ and $[f] >^\ast [g]$ respectively.

$\square$




Exercise 12.3(ii).

Solution. We mimic the proof of Lemma 17.2. If $(A,<^\ast )$ was not a well-ordering, then there exists $f_i : \kappa \to \kappa$, where $i \in \omega$, such that $[f_0] >^\ast [f_1] >^\ast \cdots$. For each $i$, denote:
$$ \begin{align*}
X_i := \lbrace \alpha \in \kappa : f_0(\alpha) > f_1(\alpha)\rbrace
\end{align*}$$ Then $X_i \in U$ for all $i$. Since $U$ is $\sigma$-complete, $X := \bigcap_{i<\omega} X_i \in U$. Then for any $\alpha \in X$, we have $f_0(\alpha) > f_1(\alpha) > \cdots$, contradicting that $\kappa$ is well-ordered.

Hence, $(A,<^\ast )$ is isomorphic to its order-type.

$\square$




Exercise 12.3(iii).

Solution. Recall from Corollary 12.5 that $j$ is an elementary embedding. Thus, we have that $j(\alpha)$ is an ordinal for all $\alpha$. Since $j$ preserves $\in$, by transfinite induction one have that $j(\alpha) \geq \alpha$ for each $\alpha$.

On the other hand, suppose $[f] < j(\alpha)$ for some $f : \kappa \to \kappa$, then $f(\beta) < \alpha$ for almost all $\beta < \kappa$. By $\alpha$-completeness (since $\alpha < \kappa$), we have that $f(\beta) = \gamma$ for some $\gamma < \alpha$. Thus, $[f] < \alpha$, and hence $j(\alpha) \leq \alpha$.

$\square$




Exercise 12.3(iv).

Solution. Given a function $f : \kappa \to \kappa$, consider the property:
$$ \begin{align}
\label{eq12.3(iv).1}
(\forall \gamma < \kappa) \lbrace \alpha : f(\alpha) > \gamma\rbrace \in U \tag{12.3(iv).1}
\end{align}$$ Note that $d(\alpha)$ is an ordinal for all $\alpha < \kappa$, so $[d]$ is an ordinal. We first note that $d$ satisfies (\ref{eq12.3(iv).1}), as $\vert \lbrace \alpha : d(\alpha) \leq \gamma\rbrace\vert < \kappa$ and $U$ is $\kappa$-complete. Thus, $j(\gamma) = \gamma < [d]$ for all $\gamma < \kappa$, and hence $[d] \geq \kappa$.

For the latter claim, Exercise 10.5 tells us that $U$ is a normal measure iff $d$ is the least such function. Since if $\kappa = [f]$ then $f$ must satisfy the property (\ref{eq12.3(iv).1}), the equivalence follows.

$\square$




Exercise 12.4.

Solution. That follows from the definition of $M$ being extensional and of $\sigma^M$.

$\square$




Exercise 12.5.

Solution. We borrow some of the formulas which are proven to be $\Delta_0$ in Lemma 12.10.

$x$ is an ordered pair:
$$ \begin{align*}
\bb{\exists y \in \bigcup x}\bb{\exists z \in \bigcup x} \, z = (x,y)
\end{align*}$$ Note that if $x = (y',z') = \lbrace \lbrace y'\rbrace,\lbrace y',z'\rbrace\rbrace$, then in the formula on RHS we may set $y = z = y'$.

$x$ is an partial ordering of $y$:
$$ \begin{gather*}
(\forall w \in y) \, (w,w) \notin x \wedge (\forall w \in y)(\forall v \in y)(\forall u \in y) \, ((w,v) \in x \wedge (v,u) \in x \to (w,u) \in x)
\end{gather*}$$ $x$ is an linear ordering of $y$:
$$ \begin{align*}
\text{$x$ is a partial ordering of $y$} \wedge (\forall w \in y)(\forall v \in y) \, (w,v) \in x \vee (v,w) \in x
\end{align*}$$ $x$ and $y$ are disjoint:
$$ \begin{align*}
x \cap y \text{ is empty}
\end{align*}$$ $z = x \cup y$:
$$ \begin{align*}
x \subseteq z \wedge y \subseteq z \wedge (\forall w \in z) \, w \in x \vee w \in y
\end{align*}$$ $y = x \cup \lbrace x\rbrace$:
$$ \begin{align*}
z \in y \leftrightarrow z \in x \vee z = x
\end{align*}$$ $x$ is an inductive set:
$$ \begin{align*}
\emptyset \in x \wedge (\forall w \in x) \, w \cup \lbrace w\rbrace \in x
\end{align*}$$ $f$ is a function of $X$ into $Y$:
$$ \begin{align*}
\text{$f$ is a function} \wedge \dom(f) = X \wedge \ran(f) \subseteq Y
\end{align*}$$ $f$ is a one-to-one function of $X$ into $Y$:
$$ \begin{align*}
\text{$f$ is a function of $X$ into $Y$} \wedge (\forall w \in X)(\forall v \in X)(f(w) = f(v) \to w = v)
\end{align*}$$ $f$ is a function of $X$ onto $Y$:
$$ \begin{align*}
\text{$f$ is a function of $X$ into $Y$} \wedge (\forall w \in X)(\exists v \in Y)(v = f(w))
\end{align*}$$ $f$ is an increasing ordinal function:
$$ \begin{align*}
\dom(f) \text{ is an ordinal} \wedge (\forall w \in \dom(f))(\forall v \in \dom(f))(w \in v \to f(w) \in f(v))
\end{align*}$$ $f$ is a normal function:
$$ \begin{gather*}
\text{$f$ is an increasing ordinal function} \\
\wedge \\
(\forall w \in \dom(f))(w \text{ is a limit ordinal} \to ((\forall v \in f(w))(\exists u \in w) \, v \in f(u)))
\end{gather*}$$

$\square$




Exercise 12.6.


Exercise 12.6(i).

Solution. We have that $M \models \vert X\vert \leq \vert Y\vert $ iff $M \models \exists f \,(f \text{ is a one-to-one function of $X$ into $Y$})$. By Exercise 12.5, $f$ remains a one-to-one function of $X$ into $Y$ in $V$, so $\vert X\vert \leq \vert Y\vert $.

$\square$




Exercise 12.6(ii).

Solution. We have $\alpha$ is a cardinal iff:
$$ \begin{align*}
\neg (\exists f \underbrace{(\exists \beta \in \alpha)\, (\text{$f$ is a function of $\alpha$ onto $\beta$})}_{\text{$\Delta_0$ formula, by Exercise 12.5}})
\end{align*} $$ Thus "$\alpha$ is a cardinal" is downward absolute, so $M \models \alpha$ is a cardinal.

$\square$




Exercise 12.7.


Lemma 12.7.A. Let $(M,\in)$ be a model, where $M \subseteq V$, and $V$ is the universe which is a model of $\ZFC$.
  1. If $(\forall x \in M)(\forall y \in M) \, \lbrace x,y\rbrace \in M$, then $M$ satisfies Pairing.
  2. If $(\forall z \in M)(\forall y \subseteq z) \, y \in M$, then $M$ satisfies Separation.
  3. $M$ satisfies Regularity.
Suppose further that $M$ is transitive. Then:
  1. $M$ satisfies Extensionality.
  2. If $(\forall X \in M) \, \bigcup X \in M$, then $M$ satisfies Union.
  3. If $(\forall x \in M) \, \Po(x) \cap M \in M$, then $M$ satisfies Power Set.
  4. If every family of non-empty sets in $M$ has a choice function in $M$, then $M$ satisfies $\AC$.
Suppose even further that $M$ satisfies the axioms Extensionality, Comprehension, Pairing, and Union. If $\omega \in M$, then $M$ satisfies Infinity.

Proof. We refer to readers to the textbook for the formal statements of each axiom.
  1. We wish to show that:
    $$ \begin{align*}
    (\forall a \in M)(\forall b \in M)(\exists c \in M)(\forall x \in M)(x \in c \leftrightarrow x = a \vee x = b)
    \end{align*}$$ Fix $a,b \in M$ and since $\lbrace a,b\rbrace \in M$ we let $c = \lbrace a,b\rbrace$. Then clearly $x \in c \leftrightarrow x = a \vee x = b$.
  2. We wish to show that:
    $$ \begin{align*}
    (\forall X \in M)(\forall p \in M)(\exists Y \in M)(\forall u \in M) \, (u \in Y \leftrightarrow u \in X \wedge \varphi(u,p))
    \end{align*}$$ For each $X \in M$ and $p \in M$, since the universe satisfies Separation there exists a $Y$ such that $\forall u \, (u \in Y \leftrightarrow u \in X \wedge \varphi(u,p))$. In particular we have $Y \subseteq X$, and since $M$ is closed under subsets we have $Y \in M$.
  3. We wish to show that:
    $$ \begin{align*}
    (\forall S \in M)(S \neq \emptyset \to (\exists x \in S) \, S \cap x = \emptyset)
    \end{align*}$$ But this is the relativised statement of a $\Pi_1$ statement, which is downward absolute. So $M$ must satisfy regularity.
  4. We wish to show that:
    $$ \begin{align*}
    (\forall X \in M)(\forall Y \in M)((\forall u \in M)(u \in X \leftrightarrow u \in Y) \to X = Y)
    \end{align*}$$ Let $X,Y \in M$. Suppose $X \neq Y$ (this sentence is absolute across models, not necessarily transitive). Then in $V$, there exists $u \in X - Y$ or $Y - X$. WLOG suppose $u \in X - Y$. Since $X \in M$ and $M$ is transitive, $u \in M$. Thus $(\exists u \in M)(u \in X \wedge \neg(u \in Y))$.
  5. We wish to show that:
    $$ \begin{align*}
    (\forall X \in M)(\exists Y \in M)(\forall u \in M)(u \in Y \leftrightarrow \exists z \in M \, (z \in X \wedge u \in z))
    \end{align*}$$ For each $X \in M$ let $Y = \bigcup X \in M$. Let $u \in M$. Since $Y$ is the candidate for union in $V$, we have $\exists z \in M \, (z \in X \wedge u \in z)$ implies $u \in Y$. Conversely, suppose $u \in Y$, so there exists some $z$ such that $z \in X$ and $u \in z$. We also need $z \in M$, but this follows from that $M$ is transitive.
  6. We wish to show that:
    $$ \begin{align*}
    (\forall X \in M)(\exists Y \in M)(\forall u \in M)(u \in Y \leftrightarrow u \subseteq X)
    \end{align*}$$ For each $X \in M$ let $Y = P(X) \cap M$. Let $u \in M$. If $u \in P(X) \cap M$, then immediately $u \subseteq X$ by definition of $P(X)$. If $u \subseteq X$, then since $u \in M$ already we have $y \in P(X) \cap M = Y$.
  7. We wish to show that if $S \in M$ is a family of sets and $\emptyset \notin S$, then there is a $f \in M$ on $S$ such that $M \models f$ is a choice function on $S$. Since there exists $f \in M$ such that $V \models f$ is a choice function on $S$, it suffices to show that "$f$ is a choice function" is downward absolute. But this is because $M$ is transitive, "$f$ is a function" is $\Delta_0$ (when $\dom(f) \in M$), and since $S \in M$ we have $(\forall X \in X) \, f(X) \in X$ is also $\Delta_0$.
  8. We wish to show that:
    $$ \begin{align*}
    (\exists S \in M)(\emptyset \in S \wedge (\forall x \in S)(x \cup \lbrace x\rbrace \in S))
    \end{align*}$$ Let $S = \omega$. Since $M$ is transitive, we have in particular that $\emptyset \in M$ and $\emptyset \in S$. Let $x \in S$. Since $M$ satisfies Pairing, we have $\lbrace x\rbrace \in M$, and $\lbrace x,\lbrace x\rbrace\rbrace \in M$. Since $M$ satisfies Union, we have $x \cup \lbrace x\rbrace \in M$. Then $x \cup \lbrace x\rbrace \in S$ in $M$ as $x \cup \lbrace x\rbrace \in S$ in $V$, and $\in$ is absolute across models (not necessarily transitive).

$\blacksquare$



Solution. We use Lemma 12.7.A.

Extensionality: This follows from that $V_\alpha$ is transitive.

Pairing: Let $a,b \in V_\alpha$, and suppose $\rank(a) = \beta < \alpha$ and $\rank(b) = \gamma < \alpha$. Then $\rank(\lbrace a,b\rbrace) = \max\lbrace \beta,\gamma\rbrace + 1 < \alpha$ as $\alpha$ is limit, so $\lbrace a,b\rbrace \in V_\alpha$.

Separation: It suffices to show that $V_\alpha$ is closed under taking subsets. But this follows from that if $X \subseteq Y$ and $Y \in V_\alpha$ then $\rank(X) \leq \rank(Y) < \alpha$ so $X \in V_\alpha$.

Union: Let $X \in V_\alpha$, so $\rank(X) < \alpha$. By Exercise 6.4 we have $\rank\bb{\bigcup X} = \bigcup \rank(x) < \alpha$, so $\bigcup X \in V_\alpha$.

Power Set: Let $X \in V_\alpha$, so $\rank(X) < \alpha$. By Exercise 6.4 again, $\rank(P(X)) = \rank(X) + 1 < \alpha$, so $P(X) \in V_\alpha$. Note furthermore that $P(X) \cap V_\alpha = P(X)$, so $P(X) \cap V_\alpha = P(X)$.

Choice: Let $S \in V_\alpha$ be a family of sets, $\emptyset \notin S$. Note that if $\rank(S) = \beta < \alpha$, then $\rank\bb{\bigcup S} = \bigcup \beta$. If in $V$ $\AC$ holds then there exists a function $f$ on $S$ such that $f(X) \in X$ for all $X \in S$. Now since $f \subseteq S \times \bigcup S$ and we have shown that $V_\alpha$ is closed under subsets, we have $f \in S \times \bigcup S$. Note that $A \times B \subseteq P(P(A \cup B))$.

$\square$




Exercise 12.8.

Solution. This follows from Lemma 12.7.A and that $\omega \in V_\alpha$.

$\square$




Exercise 12.9.

Solution. By Exercise 12.7, we're left to check Replacement. Let $X \in V_\omega$ and let $F$ be a function on $V_\omega$. In the universe $V$ we have the set $Y := \lbrace F(x) : x \in X\rbrace$. By Exercise 3.1(iv) $Y$ is finite, and since $\ran(F) \subseteq V_\omega$, $F(x)$ is finite for all $x \in X$. Thus $Y$ is hereditarily finite, and $Y \in V_\omega$. Of course $V_\omega \models Y = F(X)$.

$\square$




Exercise 12.10.

Solution. By Exercise 2.4 we have that $V_\omega$ does not satisfy Infinity as $\omega \notin V_\omega$. Thus, we have that Infinity is independent of $\ZFC$ minus Infinity.

Suppose it can be shown (with just $\ZFC$ minus Infinity) that existence of an infinite set is consistent. Then $\Con(\ZFC - \text{Infinity}) \to \Con(\ZFC)$. However, in every model $M$ of $\ZFC$, $V_\omega^M$ is a model of $\ZFC$ minus Infinity, so we have $\ZFC \vdash \Con(\ZFC - \text{Infinity})$. Thus $\ZFC \vdash \Con(\ZFC)$, contradicting Gödel's second incompleteness theorem.

$\square$




Exercise 12.11.

Solution. First note that since $\omega \in V_\kappa$ and $\omega$ is countable in all models containing it, and $E \subseteq \omega \times \omega$ so $\rank(E) \leq \omega + 2$. Thus $\A \in V_\kappa$ and $V_\kappa \models$ $\A$ is countable. To see that $V_\kappa \models$ ($\A$ is a model of $\ZFC$), we simply observe that for each axiom of $\ZFC$ $\sigma$, we have $\sigma^{\omega,E}$ is $\Delta_0$ (with all parameters in $V_\kappa$), so it is transitive between $V$ and $V_\kappa$.

$\square$




Exercise 12.12.


Lemma 12.12.A. Let $\c{M_\gamma : \gamma < \beta}$ be an increasing sequence of elementary submodels of $\c{V_\kappa,\in}$. Let $\c{\alpha_\gamma : \gamma < \beta}$ be such that $M_\gamma \subseteq V_{\alpha_\gamma}$, and suppose $\alpha := \lim_{\gamma<\beta} \alpha_\gamma < \kappa$. Then $\c{V_\alpha,\in} \prec \c{V_\kappa,\in}$.

Proof. we induct on $m$ that $M_\gamma \prec_{\Sigma_m} V_\alpha \prec_{\Sigma_m} V_\kappa$, and if $\sigma$ is a $\Sigma_m$ (or $\Pi_m$) sentence such that $M_\gamma \models \sigma$ for all $\gamma$ large enough, then $V_\alpha \models \sigma$. The case $m = 0$ is trivial, as $\Delta_0$ sentences are always absolute among transitive models. Suppose this is true for $m$.
  1. Suppose $\sigma$ is $\exists y \, \phi(x,y)$ and $V_\kappa \models \sigma$, where $\phi$ is $\Pi_m$ and $x \in V_\alpha$. Let $\gamma$ be large enough so that $x \in M_\gamma$. Then $M_\gamma \models \phi(x,y)$ as $M_\gamma \prec V_\kappa$. By induction hypothesis, $M_\gamma \prec_{\Sigma_m} V_\alpha$, so $V_\alpha \models \phi(x,y)$. Then $V_\alpha \models \exists y \, \phi(x,y)$.
  2. Suppose $\sigma$ is $\forall y \, \psi(x,y)$, where $\psi$ is $\Sigma_m$ and $x \in V_\alpha$. For any $y \in V_\alpha$, we have $V_\alpha \models \psi(x,y)$ as $V_\alpha \prec_{\Sigma_m} V_\kappa$ by induction hypothesis. Thus $V_\alpha \models \forall y \, \psi(x,y)$.

$\blacksquare$



\begin{lemma}{12.12.B}[Tarski-Vaught Criterion]

Let $\A = (A,\dots)$ and $\B = (B,\dots)$ be two structures. The following are equivalent:
  1. $\A \prec \B$.
  2. For all formulas of the form $\exists y \, \varphi(x,y)$, where $x \in A$, if $\B \models \exists y \, \varphi(x,y)$ then there exists a $y \in A$ such that $\A \models \varphi(x,y)$.

Proof. (i) $\implies$ (ii) is immediate. For (ii) $\implies$ (i), we first observe that (ii) implies that for all $\Sigma_n$ formulas $\varphi(x)$ and $x \in A$, we have $\A \models \varphi(x)$ iff $\B \models \varphi(x)$. Now suppose $\varphi(x)$ is $\Pi_n$. We induct on $n$. If $n = 0$, then $\varphi(x)$ is $\Delta_0$ and hence absolute. Suppose $\varphi(x)$ is $\Pi_{n+1}$, so we write $\varphi(x) = \exists y \, \psi(x,y)$, where $\psi$ is $\Sigma_n$. Since $\psi(x,y)$ is absolute by (ii), $\varphi(x)$ is downward absolute, so $\B \models \varphi(x)$ implies $\A \models \varphi(x)$. On the other hand, if $\B \not\models \varphi(x)$, then $\B \models \neg\varphi(x)$. But $\neg\varphi(x)$ is $\Sigma_{n+1}$, so $\A \models \neg\varphi(x)$. Hence $\A \not\models \varphi(x)$.

$\blacksquare$



Solution. There are two things which we need to show:
  1. (Unboundedness) If there exists an elementary submodel $\c{M,\in} \prec \c{V_\kappa,\in}$ and $M \subseteq V_\alpha$, then there exists $\alpha' > \alpha$ such that $\c{V_{\alpha'},\in} \prec \c{V_\kappa,\in}$.
  2. (Closedness) If $\c{\alpha_\gamma : \gamma < \beta}$, where $\beta < \kappa$, is a sequence of ordinals such that $\c{V_{\alpha_\gamma},\in} \prec \c{V_\kappa,\in}$ for all $\gamma < \beta$, then $\c{V_\alpha,\in} \prec \c{V_\kappa,\in}$ where $\alpha := \lim_{\gamma\to\beta} \alpha_\gamma$.

Closedness follows immediately from Lemma 12.12.A, by letting $M_\gamma = V_{\alpha_\gamma}$. Then $\alpha < \kappa$ as $\kappa$ is regular.

Begin with a elementary submodel $\c{M,\in} \prec \c{V_\kappa,\in}$. Let $\alpha_0 := \sup\lbrace \rank(x) + 1 : x \in M\rbrace$. Then $\rank(x) < \kappa$ for all $x \in M$, and since $\vert M\vert < \kappa$ and $\kappa$ is regular, $\alpha_0 < \kappa$. Then $M \subseteq V_{\alpha_0}$.

Now suppose $\alpha_n$ is defined. Let $\lbrace h_k : k < \omega\rbrace$ denote the set of all Skolem functions for $V_\kappa$. For each $h_k$, we note that for $x_1,\dots,x_m \in V_{\alpha_n}$, we have $h_k(x_1,\dots,x_m) \in V_{\alpha_n}$. Now:
$$ \begin{align*}
\rank(h_k(V_{\alpha_n})) = \sup\lbrace \rank(h_k(x_1,\dots,x_m)) + 1 : x_1,\dots,x_m \in V_{\alpha_n}\rbrace
\end{align*}$$ Since $\alpha_n < \kappa$, we have $\beth_{\alpha_n} < \kappa$. Thus, $\rank(h_k(V_{\alpha_n}))$ is the sup of $<\beth_{\alpha_n}$ many ordinals below $\kappa$, so $\rank(h_k(V_{\alpha_n})) < \kappa$ by regularity of $\kappa$. Therefore, there exists $\alpha_{n+1} < \kappa$ such that $\bigcup_{k=0}^\infty h_k(V_{\alpha_n}) \subseteq V_{\alpha_{n+1}}$. We may also choose $\alpha_{n+1} > \alpha_n$.

Claim. $\c{V_\alpha,\in} \prec \c{V_\kappa,\in}$.

Proof. We use Tarski-Vaught criterion (Lemma 12.12.B). Let $\exists y \, \phi(x,y)$ be an existential formula, with $x \in V_\alpha$, and suppose $V_\kappa \models \exists y \, \phi(x,y)$. Then $V_\kappa \models \phi(h_\phi(x),x)$. Now $x \in V_{\alpha_n}$ for some $n$, so $h_\phi(x) \in h(V_{\alpha_n}) \subseteq V_{\alpha_{n+1}} \subseteq V_\alpha$. Thus $V_\alpha \models \exists y \, \phi(x,y)$.

$\blacksquare$



Thus, we have proved the unboundedness property as $\alpha > \alpha_0$.

$\square$




Exercise 12.13.

Solution. In this proof we shall borrow Lemma 6.6(i).A and Lemma 12.7.A.

Extensionality: This is because $H_\kappa$ is transitive.

Pairing: Given $a,b \in H_\kappa$, we have:
$$ \begin{align*}
\vert \TC(\lbrace a,b\rbrace)\vert = \mod{\lbrace a,b\rbrace \cup \TC(a) \cup \TC(b)} \leq \vert \lbrace a,b\rbrace\vert + \vert \TC(a)\vert + \vert \TC(b)\vert < \kappa
\end{align*}$$ so $\lbrace a,b\rbrace \in H_\kappa$.

Separation: It suffices to show that $V_\alpha$ is closed under taking subsets. But this follows from that $X \subseteq Y \implies \vert \TC(X)\vert \leq \vert \TC(Y)\vert $.

Union: Let $X \in H_\kappa$. Observe that:
$$ \begin{align*}
\TC\bb{\bigcup X} &= \bb{\bigcup X} \cup \bigcup_{x \in \bigcup X} \TC(x) \\
&= \bb{\bigcup X} \cup \bigcup_{(\exists y \in X) \, x \in y} \TC(x) \\
&= \bb{\bigcup_{y \in X} y} \cup \bigcup_{y \in X} \bigcup_{x \in y} \TC(x) \\
&= \bigcup_{y \in X} \bb{y \cup \bigcup_{x \in y} \TC(x)} \\
&= \bigcup_{y \in X} \TC(y)
\end{align*}$$ Now $\vert \TC(y)\vert < \kappa$ for all $y \in X$. Since $\vert X\vert < \kappa$, we have that $\mod{\bigcup_{y \in X} \TC(y)} < \kappa$. Hence $\bigcup X \in H_\kappa$.

Infinity: Clearly $\omega \in H_\kappa$.

Replacement: Let $F$ be a function on $H_\kappa$ into $H_\kappa$, and let $X \in H_\kappa$. We have:
$$ \begin{align*}
\TC(F(X)) = \lbrace F(x) : x \in X\rbrace \cup \bigcup_{x \in X} \TC(F(x))
\end{align*}$$ Since $F(x) \in H_\kappa$ for all $x \in H_\kappa$, $\vert \TC(F(x))\vert < \kappa$. Note also that $\vert F(X)\vert \leq \vert X\vert < \kappa$, and therefore $\vert \TC(F(X))\vert < \kappa$. Therefore $\TC(F(X)) \in H_\kappa$.

Regularity: Immediate from Lemma 12.7.A.

Choice: Let $S \in H_\kappa$ be a family of sets, $\emptyset \notin S$. Let $f$ be a choice function. Observe that:
$$ \begin{align*}
\TC(f) &= f \cup \bigcup_{x \in \dom(f)} \TC((x,f(x)))
\end{align*}$$ For each $x \in \dom(f)$, we have:
$$ \begin{align*}
\TC((x,f(x)) &= \TC(\lbrace \lbrace x\rbrace,\lbrace x,f(x)\rbrace) \\
&= \lbrace \lbrace x\rbrace,\lbrace x,f(x)\rbrace\rbrace \cup \TC(\lbrace x\rbrace) \cup \TC(\lbrace x,f(x)\rbrace) \\
&= \lbrace \lbrace x\rbrace,\lbrace x,f(x)\rbrace\rbrace \cup \TC(x) \cup \TC(f(x))
\end{align*}$$ Since $x,f(x) \in H_\kappa$, we have $\vert \TC(x)\vert < \kappa$ and $\vert \TC(f(x))\vert < \kappa$ for all $x \in \dom(f)$. Thus $\vert \TC((x,f(x)))\vert < \kappa$. Furthermore, we have that $\vert f\vert = \vert \dom(f)\vert < \kappa$, and since $\kappa$ is regular we have that $\mod{\bigcup_{x \in \dom(f)} \TC((x,f(x)))} < \kappa$. Therefore $\vert \TC(f)\vert < \kappa$, so $f \in H_\kappa$.

$\square$




Exercise 12.14.

Solution. We induct on the complexity of $\varphi$. If $\varphi$ is atomic (i.e. $x \in y$ or $x = y$), then they are in particular $\Delta_0$ and hence is reflected by all of $V_\alpha$, where $\alpha \geq \omega$. As in the hint, we have $C_{\varphi \wedge \psi} = C_\varphi \cap C_\psi$. Clearly $C_{\neg\varphi} = C_\varphi$.

We shall now show that for all $\alpha \in C_\varphi \cap K_\varphi$, then $V_\alpha$ reflects $\exists x \, \varphi$. Note that this does not imply that all such $\alpha$'s are in $C_\varphi \cap K_\varphi$ - we simply claim that there exists a closed unbounded subclass of such $\alpha$'s.

$K_\psi$ is closed unbounded: We note that unboundedness follows from Reflection Principle (Theorem 12.14(i)), as $M_0$ may be chosen to be arbitrarily large. To see that it is closed, let $\lbrace \alpha_\gamma : \gamma < \beta\rbrace \subseteq K_\psi$ and let $\alpha := \lim_{\gamma\to\beta} \alpha_\gamma$. Let $x_1,\dots,x_n \in V_\alpha$, and suppose $\exists x \, \varphi(x,x_1,\dots,x_n)$. If $x_i \in V_{\alpha_{\gamma_i}}$, then $x_1,\dots,x_n \in V_{\alpha'}$ where $\alpha' := \max\lbrace \alpha_1,\dots,\alpha_n\rbrace$. Since $\alpha' \in K_\psi$, we have $(\exists x \in V_{\alpha'}) \, \varphi(x,x_1,\dots,x_n)$ holds. Thus $(\exists x \in V_\alpha) \, \varphi(x,x_1,\dots,x_n)$ holds as $V_{\alpha'} \subseteq V_\alpha$.

Let $\alpha \in C_\psi \cap K_\psi$. Suppose $\exists x \, \psi(x,x_1,\dots,x_n)$ holds, where $x_1,\dots,x_n \in V_\alpha$. Since $\alpha \in K_\psi$, there exists $x' \in V_\alpha$ such that $\psi(x',x_1,\dots,x_n)$ holds. Since $\alpha \in C_\psi$, $\psi$ is reflected so $\psi^{V_\alpha}(x',x_1,\dots,x_n)$ holds. Therefore $(\exists x \, \psi)^{V_\alpha}$ holds.

Conversely suppose $V_\alpha \models \exists x \, \psi(x,x_1,\dots,x_n)$, where $x_1,\dots,x_n \in V_\alpha$. Let $x' \in V_\alpha$ such that $\psi^{V_\alpha}(x',x_1,\dots,x_n)$. Since $\alpha \in C_\psi$ we have that $\psi(x',x_1,\dots,x_n)$ holds, and therefore $\exists x \, \psi(x,x_1,\dots,x_n)$ holds.

Therefore, $V_\alpha$ reflects $\exists x \, \psi$.

$\square$




Exercise 12.15.

Solution. We may make small modifications to the proof of Theorem 12.15 by letting, in (12.26):
$$ \begin{align*}
C := \lbrace x \in M : \varphi(u_1,\dots,u_n,x)\rbrace
\end{align*}$$ Then $M_i$'s constructed later remain subsets of $M$ by transitivity. The rest of the proof is verbatim.

$\square$




Exercise 12.16.

Solution. Again we may modify the proof of Theorem 12.15. We let, in (12.26):
$$ \begin{align*}
C := \lbrace x \in W : \varphi(u_1,\dots,u_n,x)\rbrace
\end{align*}$$ Replace each $M_i$ with $W_\alpha$, where $\alpha$ is the least ordinal such that $M_i \subseteq W_\alpha$. The rest of the proof passes again by transitivity of $W$.

$\square$